How to fully solve this limit evaluation using integration?

  • MHB
  • Thread starter juantheron
  • Start date
  • Tags
    Limit
In summary, the problem is solved by defining a function and using its derivative to show that it is close to x when x is small. Then using that fact, we can get an upper and lower bound for the sum and take the limit to get the answer. This can also be solved using an integral.
  • #1
juantheron
247
1
Evaluation of \(\displaystyle \displaystyle \lim_{n\rightarrow \infty}\sum^{n}_{k=1}\bigg(\frac{k}{n^2}\bigg)^{\frac{k}{n^2}+1}\)
 
Mathematics news on Phys.org
  • #2
jacks said:
Evaluation of \(\displaystyle \displaystyle \lim_{n\rightarrow \infty}\sum^{n}_{k=1}\bigg(\frac{k}{n^2}\bigg)^{\frac{k}{n^2}+1}\)

We can set the followings:
\(\displaystyle
dx=\frac{1}{n}\)
and
\(\displaystyle
x=\frac{k}{n}
\)
As \(\displaystyle n\rightarrow \infty\) \(\displaystyle \sum\) is replaced with \(\displaystyle \int\).
So, we finally have:

\(\displaystyle
\int_{0}^{1} (x dx)^{1 +x dx}
=> \int_{0}^{1} x dx ((x dx)^{x})^{dx}
\)

Term inside () in above integration will be one because in limiting case \(\displaystyle dx \rightarrow 0.\) To be honest I might be taking a leap here. :)
So, the problem actually reduces to:

\(\displaystyle
\int_{0}^{1} x dx
\)

Whose value is 0.5.
 
Last edited by a moderator:
  • #3
Satya said:
jacks said:
Evaluation of \(\displaystyle \displaystyle \lim_{n\rightarrow \infty}\sum^{n}_{k=1}\bigg(\frac{k}{n^2}\bigg)^{\frac{k}{n^2}+1}\)
We can set the followings:
\(\displaystyle
dx=\frac{1}{n}\)
and
\(\displaystyle
x=\frac{k}{n}
\)
As \(\displaystyle n\rightarrow \infty\) \(\displaystyle \sum\) is replaced with \(\displaystyle \int\).
So, we finally have:

\(\displaystyle
\int_{0}^{1} (x dx)^{1 +x dx}
=> \int_{0}^{1} x dx ((x dx)^{x})^{dx}
\)

Term inside () in above integration will be one because in limiting case \(\displaystyle dx \rightarrow 0.\) To be honest I might be taking a leap here. :)
So, the problem actually reduces to:

\(\displaystyle
\int_{0}^{1} x dx
\)

Whose value is 0.5.
Brilliant intuition, Satya! But this is a math forum, not an engineering forum, so your argument needs a few sticking plasters to make it rigorous.

[sp]Define a function $f(x)$ for $x\geqslant0$ by $$f(x) = \begin{cases}x^{x+1}&(x>0),\\0&(x=0). \end{cases}$$ Then $f$ is continuously differentiable, with $$f'(x) = \begin{cases}x^x(x + \ln x + 1)&(x>0),\\ 1&(x=0).\end{cases}$$ Strictly speaking, the derivative at $x=0$ is only a one-sided derivative. But that is the crucial fact that is needed, because it implies that $f(x)$ is very close to $x$ when $x$ is small. More precisely, given $\varepsilon>0$ there exists $\delta>0$ such that $(1-\varepsilon)x < f(x) < (1+\varepsilon)x$ whenever $0<x<\delta$.

Now choose $n$ with $\frac1n<\delta$, and let $1\leqslant k\leqslant n$. We can then put $x = \frac k{n^2}$ in the above inequalities to get $$(1-\varepsilon)\frac k{n^2} < \bigg(\frac{k}{n^2}\bigg)^{\frac{k}{n^2}+1} < (1+\varepsilon)\frac k{n^2}.$$ Sum that from $k=1$ to $n$, using the fact that \(\displaystyle \sum_{k=1}^n k = \tfrac12n(n+1)\), getting $$(1-\varepsilon)\frac{n(n+1)}{2n^2} < \sum_{k=1}^n \bigg(\frac{k}{n^2}\bigg)^{\frac{k}{n^2}+1} < (1 + \varepsilon)\frac{n(n+1)}{2n^2}.$$ Let $n\to\infty$ to get $$\frac12(1-\varepsilon) \leqslant \lim_{n\to\infty} \sum_{k=1}^n \bigg(\frac{k}{n^2}\bigg)^{\frac{k}{n^2}+1} \leqslant \frac12(1+\varepsilon).$$ Finally, let $\varepsilon\to0$ to see that \(\displaystyle \lim_{n\to\infty} \sum_{k=1}^n \bigg(\frac{k}{n^2}\bigg)^{\frac{k}{n^2}+1} = \frac12\).

(The last part of that argument could alternatively be done by using a Riemann sum approximation to an integral, which is what Satya was doing.)

[/sp]
 
Last edited:
  • Like
Likes juantheron
  • #4
Wow. I had struggled to formalize my solution. You solved this problem in the way it should be solved. Great.
 
  • #5
Thank you Opalg. I was struggling to find a rigorous proof as well.
Using your solution, I could finish mine.
Just as a slight alternative, here is my solution.

Let's take as a given that:
$$\lim_{x\to 0^+} x^x = 1 \tag{1}$$
We can prove it separately (indirectly) with l'Hôpital's rule, but I'll keep that out of scope for now.

It means that for every $\varepsilon >0$ there is an $\delta >0$ such that for every $0<x<\delta$ we have: $1-\varepsilon < x^x < 1 + \varepsilon$.
Moreover, for $N$ sufficiently big we have that $0 < \frac 1N < \delta$.
That is, there is an $N$ such that for all $n> N$ and for all $1\le k \le n$ we have that $0<\frac k{n^2} \le \frac n{n^2} < \frac 1N < \delta$.
And therefore:
$$1-\varepsilon < \left(\frac k{n^2}\right)^{\frac k{n^2}} < 1 + \varepsilon \tag{2}$$

Let $s_n$ be the summation in the problem statement up to $n$. Then:
\begin{aligned}
s_n = \sum^{n}_{k=1}\bigg(\frac{k}{n^2}\bigg)^{\frac{k}{n^2}+1}
&= \left(\frac 1{n^2}\right)^{\frac 1{n^2}+1} + \left(\frac 2{n^2}\right)^{\frac 2{n^2}+1} + \ldots + \left(\frac n{n^2}\right)^{\frac n{n^2}+1} \\
&= \frac 1{n^2}\left[ \left(\frac 1{n^2}\right)^{\frac 1{n^2}} + \left(\frac 2{n^2}\right)^{\frac 2{n^2}}\cdot 2 + \ldots + \left(\frac n{n^2}\right)^{\frac n{n^2}} \cdot n \right]
\end{aligned}
Using (2), we get the following.
For every $\varepsilon >0$ there is an $N$ such that for all $n> N$:
\begin{aligned}\frac 1{n^2}\big[ (1-\varepsilon) + (1-\varepsilon)2 + \ldots + (1-\varepsilon)n \big]
&< s_n < \frac 1{n^2}\big[ (1+\varepsilon) + (1+\varepsilon)2 + \ldots + (1+\varepsilon)n \big] \\
\frac {1-\varepsilon}{n^2}\cdot \frac 12n(n+1) &< s_n < \frac {1+\varepsilon}{n^2}\cdot \frac 12n(n+1) \\
\frac 12 (1-\varepsilon) &< s_n < \frac 12(1+\varepsilon)(1+\frac 1n)
\end{aligned}
Now let $\varepsilon \to 0^+$ and $n\to\infty$ and we get:
$$\lim_{n\to\infty}\sum^{n}_{k=1}\bigg(\frac{k}{n^2}\bigg)^{\frac{k}{n^2}+1} = \lim_{n\to\infty} s_n = \frac 12$$
 
  • Like
Likes juantheron
  • #6
One more variation. This time with an integral as Satya suggested and with sticking plasters from Opalg.

Let $s_n = \sum^{n}_{k=1}\left(\frac{k}{n^2}\right)^{\frac{k}{n^2}+1}$.
Let $x_k = \frac kn$ and $\Delta x = \frac 1n$.

Then it follows from the definition of a Riemann integral that:
$$\lim_{n\to\infty}\sum_{k=1}^n x_k\Delta x = \int_0^1 x\,dx \tag 1$$

Using $\lim\limits_{x\to 0^+} x^x = 1$ we can find the following, as explained in my previous solution.

For every $\varepsilon > 0$ there is an $N$ such that for all $n>N$ and all $1\le k \le n$:
$$1-\varepsilon < (x_k \Delta x)^{x_k \Delta x} < 1+\varepsilon \tag 2$$

We have:
$$
s_n =\sum^{n}_{k=1}\bigg(\frac{k}{n^2}\bigg)^{\frac{k}{n^2}+1}
= \sum^{n}_{k=1}(x_k\Delta x)^{x_k\Delta x+1}
= \sum^{n}_{k=1}x_k\Delta x(x_k\Delta x)^{x_k\Delta x}
$$

Therefore, using (2) for $n>N$:
$$
\sum^{n}_{k=1}x_k\Delta x(1-\varepsilon) < s_n < \sum^{n}_{k=1}x_k\Delta x(1+\varepsilon)\\
(1-\varepsilon)\lim_{n\to\infty}\sum^{n}_{k=1}x_k\Delta x \le \lim_{n\to\infty} s_n \le (1+\varepsilon)\lim_{n\to\infty}\sum^{n}_{k=1}x_k\Delta x\\
$$
With (1) we get:
$$(1-\varepsilon) \int_0^1 x\,dx \le \lim_{n\to\infty}s_n \le (1+\varepsilon) \int_0^1 x\,dx$$
which holds true for every $\varepsilon>0$.

Thus:
$$\lim_{n\to\infty} s_n = \int_0^1 x\,dx = \frac 12$$
 
  • Like
Likes juantheron
  • #7
Klaas van Aarsen said:
One more variation. This time with an integral as Satya suggested.

Let $s_n = \sum^{n}_{k=1}\left(\frac{k}{n^2}\right)^{\frac{k}{n^2}+1}$.
Let $x_k = \frac kn$ and $\Delta x = \frac 1n$.

Then it follows from the definition of a Riemann integral that:
$$\lim_{n\to\infty}\sum_{k=1}^n x_k\Delta x = \int_0^1 x\,dx \tag 1$$

Using $\lim\limits_{x\to 0^+} x^x = 1$ we can find the following, as explained in my previous solution.

For every $\varepsilon > 0$ there is an $N$ such that for all $n>N$ and all $1\le k \le n$:
$$1-\varepsilon < (x_k \Delta x)^{x_k \Delta x} < 1+\varepsilon \tag 2$$

We have:
$$
s_n =\sum^{n}_{k=1}\bigg(\frac{k}{n^2}\bigg)^{\frac{k}{n^2}+1}
= \sum^{n}_{k=1}(x_k\Delta x)^{x_k\Delta x+1}
= \sum^{n}_{k=1}x_k\Delta x(x_k\Delta x)^{x_k\Delta x}
$$

Therefore, using (2) for $n>N$:
$$
\sum^{n}_{k=1}x_k\Delta x(1-\varepsilon) < s_n < \sum^{n}_{k=1}x_k\Delta x(1+\varepsilon)\\
(1-\varepsilon)\lim_{n\to\infty}\sum^{n}_{k=1}x_k\Delta x \le \lim_{n\to\infty} s_n \le (1+\varepsilon)\lim_{n\to\infty}\sum^{n}_{k=1}x_k\Delta x\\
$$
With (1) we get:
$$(1-\varepsilon) \int_0^1 x\,dx \le \lim_{n\to\infty}s_n \le (1+\varepsilon) \int_0^1 x\,dx$$
which holds true for every $\epsilon>0$.

Thus:
$$\lim_{n\to\infty} s_n = \int_0^1 x\,dx = \frac 12$$

This solution in my view is the simplest and still rigorous.
 
  • #8
Thanks friends for yours fantastic solutions

i have solved it using Integration.

after seeing above solutions by opalg and klass van have seems that my solution is partial (Not fully satisfactory)
 

1. What is a limit evaluation?

A limit evaluation is a mathematical concept that involves finding the value that a function approaches as its input approaches a certain value. It is often used in calculus to determine the behavior of a function near a specific point.

2. How does integration help solve limit evaluations?

Integration is a mathematical process that allows us to find the area under a curve. By using integration, we can determine the exact value that a function approaches as its input approaches a certain value, which is the limit evaluation.

3. What are the steps to fully solve a limit evaluation using integration?

The steps to fully solve a limit evaluation using integration are as follows:

  1. Determine the function and the value that the input is approaching.
  2. Find the indefinite integral of the function.
  3. Substitute the value that the input is approaching into the indefinite integral.
  4. Simplify the resulting expression.
  5. Take the limit of the simplified expression as the input approaches the given value.

4. Are there any special cases when using integration to solve limit evaluations?

Yes, there are some special cases when using integration to solve limit evaluations. One common case is when the function being evaluated has a discontinuity at the given value. In this case, we must take the limit from both sides of the discontinuity separately.

5. Can limit evaluations be solved without using integration?

Yes, limit evaluations can be solved using other methods such as algebraic manipulation or L'Hopital's rule. However, integration is often the most efficient method for solving limit evaluations, especially for more complex functions.

Similar threads

Replies
3
Views
1K
Replies
2
Views
1K
Replies
14
Views
1K
  • General Math
Replies
4
Views
1K
  • General Math
Replies
1
Views
276
  • Math POTW for University Students
Replies
3
Views
603
  • General Math
Replies
3
Views
817
Replies
2
Views
1K
  • General Math
Replies
10
Views
862
Replies
4
Views
430
Back
Top